LSAT and Law School Admissions Forum

Get expert LSAT preparation and law school admissions advice from PowerScore Test Preparation.

 Administrator
PowerScore Staff
  • PowerScore Staff
  • Posts: 8916
  • Joined: Feb 02, 2011
|
#41622
Complete Question Explanation
(The complete setup for this game can be found here: lsat/viewtopic.php?t=15887)

The correct answer choice is (C)

The approach to Global, Cannot Be True questions invariably depends, at least in part, on the nature of the supplied answer choices. Sometimes, all it takes is using an inference already made (such as a Not Law). Unfortunately, this is not the case here, as each answer choice proposes a Block of two consecutive variables. Our job is to determine which answer choice contains a Not-block, i.e. a block that cannot be true given the relationships established by three rules.

Answer choice (A): This answer choice is incorrect, because F could be fourth, and J—fifth without violating any of the rules. See Template 1.

Answer choice (B): This answer choice is incorrect, because a GH block can be created in several solutions to the game, as long as F performs immediately after H (i.e., a GHF block). In fact, you can use the local diagram for Question #2 to eliminate this answer choice relatively quickly—such shortcuts can save you valuable time in the long run.
A GH block is possible in either of the two templates, which is yet another route to eliminating answer choice (B).

Answer choice (C): This is the correct answer choice, because—thanks to the first and third rules—G must perform earlier than both H and F. Consequently, H cannot perform immediately before G.

Answer choice (D): This answer choice is incorrect, because a JG block can be formed as shown in the local diagram for Question #2. As mentioned earlier, do not let physical proximity on the main diagram influence your evaluation of the relationships between variables,. Just because J appears to the right of G does not mean that J must be later than G. The relationship between G and J is unknown.

Another way to eliminate answer choice (D) without creating a new diagram is to use your templates. While Template 1 does not allow for a JG block, Template 2 does (as long as K performs first).

Answer choice (E): This answer choice is incorrect, because a KH block is possible as long as G performs first:
PT65_D11 LG Explanations_game_#1_#3_diagram 1.png
The KH block is also validated by Template 1, making your task of eliminating answer choice (E) even simpler.
You do not have the required permissions to view the files attached to this post.

Get the most out of your LSAT Prep Plus subscription.

Analyze and track your performance with our Testing and Analytics Package.